View Single Post
 
Reply
Posted 2015-04-20, 01:02 AM in reply to Chruser's post starting "The integral seems fine for d=1 (both..."
You are correct, again. I'm lost at this point. Don't know what's wrong. Giving up for now.

Thanks.
Old
Profile PM WWW Search
Demosthenes seldom sees opportunities until they cease to beDemosthenes seldom sees opportunities until they cease to beDemosthenes seldom sees opportunities until they cease to beDemosthenes seldom sees opportunities until they cease to be
 
Demosthenes